Pediatric Orthopaedic MCQS ONLINE BANK

(SAE09TR.72) A 13-year-old girl sustained an isolated midshaft left femoral fracture in a motor vehicle accident. The fracture was treated with a rigid, antegrade intramedullary nail placed through the piriformis fossa. The fracture healed uneventfully, as shown in Figure 46a; however, at 12 months postoperatively she now reports left hip pain. A current AP radiograph and MRI scan are shown in Figures 46b and 46c. What complication occurred in this patient? Review Topic

 

 

 

  1. Chondrolysis

  2. Ischemic necrosis of the femoral head

  3. Ischemic necrosis of the greater trochanter

  4. Femoral neck fracture

  5. Trochanteric overgrowth

 

PREFERRED RESPONSE 2

 

The development of femoral head ischemic necrosis is the iatrogenically created complication in this skeletally immature patient. Placement of a rigid, antegrade intramedullary nail through the piriformis fossa is likely to damage the vascular supply to the femoral head as the vessels ascend the femoral neck on the way to the femoral head. The MRI scan reveals ischemic necrosis with early collapse of the femoral head. The joint space is preserved on the MRI scan, ruling out chondrolysis.

 

(SAE08OS.196) An adult patient has had low-grade fevers, malaise, and joint pain for the past several days. Examination reveals an erythematous "bulls eye" rash over the right thigh (erythema migrans). What is the organism associated with this infection? Review Topic

  1. Borrelia burgdorferi

  2. Bartonella henselae

  3. Nocardia

  4. Kingella kingae

  5. Brucella

 

PREFERRED RESPONSE 1

 

Erythema migrans is typically found in patients with Lyme's disease, which is caused by Borrelia burgdorferi and is usually transmitted by ticks. Bartonella henselae is associated with cat-scratch fever. Kingella kingae can be a cause of pediatric osteomyelitis; whereas, Nocardia and Brucella cause mono-articular forms of septic arthritis in adults, and are not associated with erythema migrans.

 

(SAE10PE.53) Figures 53a through 53d show the clinical photographs and radiographs of the lower extremity of a newborn male. Examination reveals this to be an isolated finding. The child otherwise has a normal neurologic examination. The hips are stable and there are no spinal defects. What is the most appropriate treatment at this time? Review Topic

 

 

  1. Symes amputation once ambulatory

  2. Observation as the deformity will slowly resolve and the child will be left with a limb-length discrepancy

  3. Immediate osteotomy for correction of the deformity

  4. Casting for correction of the deformity

  5. Genetic testing for neurofibromatosis

 

PREFERRED RESPONSE 2

 

The radiographs and clinical photographs reveal a child with posteromedial bowing of the tibia. This is a congenital anomaly that is associated with a calcaneal valgus foot. It is a relatively benign condition. The severity of the bow diminishes with time; however, the child will be left with a limb-length discrepancy, usually in the range of

4 cm. The residual limb-length discrepancy presents the greatest challenge for orthopaedic management. This, however, can usually be handled with limb-lengthening techniques. Casting can be used for severe cases with unresolving significant contracture; however, gradual spontaneous correction is usually the norm. This condition is quite different from anterior lateral bowing that can be associated with neurofibromatosis and pathologic fracture or pseudoarthrosis of the tibia.

(SAE13PE.59) Figures 59a and 59b are the radiographs of a 7-year-old boy who was seen 1 week after he underwent a closed reduction and casting in the emergency department after a fall on an outstretched arm. What is the most appropriate next step for this patient? Review Topic

 

 

 

  1. Observation

  2. Repeat closed reduction and casting

  3. Open reduction and plate fixation

  4. Closed reduction and intramedullary nail fixation

 

PREFERRED RESPONSE 1

 

This child's radiograph shows an acceptably reduced fracture of both the radius and ulna. Generally accepted limits of shaft angulation for cast treatment for girls 8 years of age or younger and boys age 10 or younger are 20 degrees for distal-third, 15 degrees for middle-third, and 10 degrees for proximal-third fractures. Remodeling decreases as one goes from distal to proximal in the forearm. Unless the child's fracture deviates from these criteria, surgical treatment is not necessary. Because of the risk of displacement, however, close follow-up is recommended.

 

(OBQ13.91) An 8-year-old sustains the injury shown in Figures A and B. Postoperative radiographs are shown in Figures C and D. After cast removal and in-office k-wire removal, elbow range of motion is found to be between 20-80 degrees of flexion. How soon after surgery is range of motion expected to be >90% of normal? Review Topic

 

 

  1. 2 months after surgery

  2. 3 months after surgery

  3. 6 months after surgery

  4. 9 months after surgery

  5. 12 months after surgery

 

PREFERRED RESPONSE 3

 

After closed reduction and percutaneous pinning of a displaced pediatric supracondylar fracture, it will take an average of 6 months for 94% of normal elbow range of motion to return.

 

Displaced pediatric supracondylar elbow fractures are often treated with closed reduction and percutaneous pinning. Risk of stiffness is low considering the extra-articular nature of the injury. Utility of physical therapy to improve range of motion after operative treatment of this injury is not supported in the literature.

 

Zionts et al. retrospectively evaluated elbow stiffness after treatment for pediatric supracondylar fractures. Ninety-four percent of normal range of motion was restored by 6 months after the operative procedure. Further improvement occurs up to one-year postoperatively

 

Lee et al. surveyed 76 orthopaedic surgeons on their treatment of pediatric supracondaylar injuries. They noted that general orthopaedic surgeons used passive exercises to assist with elbow stiffness after operative management of pediatric supracondylar fractures. In contrast, pediatric orthopaedists used active range of motion to improve elbow motion.

 

Keppler et al. retrospectively reviewed patients who had sustained supracondaylar fractures to ascertain the effect of postoperative physical therapy. At one year follow up, there was no difference in motion between patients who had received physical therapy and those who had not.

 

Figures A and B demonstrate the AP and lateral radiographs of a pediatric supracondylar fracture. Figures C and D are the postoperative radiographs after closed reduction and percutaneous pinning of the elbow injury.

Incorrect Answers:

Answers 1, 2, 4, 5: Range of motion is greater than 90% at six months after the closed reduction and percutaneous pinning procedure.

 

(SBQ13PE.13) A 17-month-old boy is referred to your clinic by his pediatrician for a workup of "bowed legs." The patient's mother states that she has other children that grew out of it by age 3 or 4. Examination of the patient reveals a child whose measurements reside in the 75% percentile for height and weight. His gait appears appropriate for a toddler without varus thrust. Supine measurements show 10-degrees of symmetric, bilateral genu varus. Appropriate workup includes: Review Topic

 

  1. Longterm clinical and radiographic monitoring to plan for guided growth until skeletal maturity

  2. X-rays today and at age 4 to assure resolution of his genu varum

  3. Genetic screening for metabolic causes of Rickets

  4. Vitamin D levels drawn today and weekly 50,000 IU vitamin D injections assuming the value is low

  5. Clinical observation and follow up to monitor resolution of genu varum

 

PREFERRED RESPONSE 5

 

This patient has physiologic genu varum or tibia vara, which is common in toddlers less than 2 years of age. The mainstay of treatment is simple observation, as there is predictable resolution of this alignment as children

 

There are numerous etiologies of genu varum in children. Most common in the children under age 2 is a predictable pattern of genu varum and internal torsion in the first year or so after beginning to walk. Persistence of genu varum, severe (>20 degrees) or worsening deformity, limb asymmetry or varus thrust gait, and low height and weight percentiles should trigger further workup with x-rays and metabolic or genetic screening. A family history of genu varum is non-specific, and in isolation, does not indicate heritable causes of genu varum.

 

Kling and Hensinger's review article highlights normal angular and torsional deviations in growing children, including genu varum and genu valgum as well as hip and hind foot deformities. They emphasize that understanding the natural history of these deformities in growing children helps the treating physician identify those who need more than simple observation of their malalignment.

 

Staheli et al. published normal values of rotational profile physical exam maneuvers (including thigh foot axis, transmalleolar axis, prone internal and external hip rotation) to better understand the predictable changes in different age groups.

Birch provides an update on current treatment of Blount's disease. He suggests further radiographic evaluation in ambulatory infants with asymmetric varus deformity or varus deformity that persists after age 18 months. If these findings are present, radiographs are recommended. If the X-rays reveal metaphyseal-diaphyseal angle

>16° on AP radiographs and confirm the diagnosis of infantile Blount disease, he recommends anti-varus long leg bracing during ambulation for patients aged <=3 years with progressive deformity, clear radiographic evidence of infantile Blount disease, or lateral thrust with ambulation.

 

Illustration A shows pathologic metaphyseal beaking common in Blount's disease. There are no pathognomonic x-ray findings in physiological bowing as presented in this question.

 

Incorrect Answers:

Answer 1: This would be appropriate for pathologic causes of tibia vara or genu varum, such as Blount's disease Answer 2: X-rays are not indicated in this age group (< 2 years old) in the absence of an abnormal history or worrisome exam findings Answer 3: This patient is unlikely to have Rickets based on presentation and age; this would be appropriate if the deformity persists or worsens overtime Answer 4: A workup for nutritional Vitamin D deficiency may be appropriate if the patient was under the 20th percentile in growth curves

 

(SAE13PE.71) Figure 71 is the MRI scan of a 2-year-old girl who has been febrile for 1 week and has refused to bear weight on her left lower extremity for 3 days. Her entire left lower extremity is markedly swollen. Doppler ultrasound shows a deep venous thrombosis of the internal iliac vein. Her white blood cell count is 19000/ µL (reference range, 4500-11000/ µL) and her C-reactive protein level is higher than 20 mg/L (reference range, 0.08-3.1 mg/L). If blood cultures yield positive results, what is the most likely organism? Review Topic

 

 

 

  1. Methicillin-resistant Staphylococcus aureus

  2. Salmonella typhii

  3. Escherichia coli

  4. Vancomycin-resistant Enterococcus

 

PREFERRED RESPONSE 1

 

The clinical picture is one of infection and deep venous thrombosis. The MRI scan is consistent with osteomyelitis. Deep venous thrombosis in association with musculoskeletal infection is more common in osteomyelitis caused by methicillin-resistant Staphylococcus aureus. Presenting C-reactive protein levels generally are higher than 6 mg/L and are higher than with other causative organisms. The presence of the Panton-Valentine leukocidin gene encoded in strains of bacteria may explain the deep venous thrombosis.

 

(SAE13PE.2) What gene is implicated in spinal muscular atrophy? Review Topic

 

  1. Survival motor neuron I (SMN-I)

  2. Peripheral myelin protein 22 (PMP22)

  3. Dystrophin

  4. Androgen receptor

 

PREFERRED RESPONSE 1

 

Deletions in the SMN-I gene are found in 95% to 98% of patients with spinal muscular atrophy. Genetic testing is typically part of the diagnostic workup for spinal muscular atrophy. A positive test result is diagnostic, and, in most cases, eliminates the need for muscle biopsy. The other choices are not associated with spinal muscular atrophy. Defects in PMP22 are the cause of 70% to 80% of cases of Charcot-Marie-Tooth disease. Mutations in the dystrophin gene cause Duchenne muscular dystrophy, and mutations in the androgen receptor cause spinobulbar muscular atrophy (Kennedy's disease).

(SAE13PE.73) A 13-year-old boy was evaluated for leg length difference. His pelvis balanced when a 1-inch (2.54 cm) block was placed under his left foot. History revealed he had a left distal femur physeal fracture treated with casting at age 10. Radiographs show normal limb alignment, but his left distal femoral physis is closed and his left femur is 2.5 cm shorter than the right. All other physes are open. His bone age is equal to his chronologic age. What surgical treatments will best equalize his discrepancy? Review Topic

 

  1. Right distal femoral and proximal tibia/fibula epiphysiodesis

  2. Right distal femoral epiphysiodesis

  3. Right proximal tibia/fibula epiphysiodesis

  4. Left proximal tibia/fibula epiphysiodesis

 

PREFERRED RESPONSE 1

 

Because the left distal femoral physis is closed with a leg length difference already at 1 inch, epiphysiodesis of both the right distal femur and proximal tibia/fibula is needed. The amount of correction will be the amount of growth remaining in the left proximal tibia. Presuming the standard rates of growth of 10 mm per year distal femur, 6 mm per year proximal tibia, and 4 mm per year distal tibia, this should yield a correction of 6 mm x 3 years = 1.8 cm by skeletal maturity at age 16. This would leave the boy with an acceptable discrepancy of 7 mm, well under 1 inch/2.54 cm. Closing only the right distal femoral physis will leave the discrepancy unchanged at 1 inch/2.54 cm because no growth differential will exist. Closing the right proximal tibia/fibular physis would mean the left knee will grow at 6 mm per year, but the right will grow at 10 mm per year. The discrepancy would increase by 4 mm per year, or

1.2 cm by skeletal maturity of age 16, leaving the boy 3.7 cm short on the left. Closing the left proximal tibia/fibula physis would increase the discrepancy at the rate of 1.6 cm per year x 3 years because both the right femoral and proximal tibia/fibular physes would be growing, leaving the boy 4.8 + 2.5 = 7.3 cm shorter on the left leg.

 

(SAE13PE.18) A displaced pediatric supracondylar humerus fracture is treated with closed manipulation and placement of 2 Kirschner wires placed from the lateral side. What would be the effect of adding a third pin from the lateral side? Review Topic

 

  1. Increase risk for iatrogenic ulnar nerve injury

  2. Provide more construct stiffness than adding a medial pin

  3. Improve construct stiffness in the presence of medial column comminution

  4. Will not affect construct stiffness in the presence of residual distal fragment internal rotation

 

PREFERRED RESPONSE 3

Multiple biomechanical studies have shown that the addition of a third pin from the lateral side improves construct stiffness in the presence of medial column loss or slight internal rotation of the distal fragment. The same studies show that addition of a medial pin (cross pinning) has essentially the same benefit. Placement of a medial pin increases risk for iatrogenic nerve injury.

 

(OBQ13.172) A 6-year-old sustains the injury shown in Figures A and B. The nerve most commonly affected by this fracture pattern innervates which of the following motor groups?

Review Topic

 

 

 

  1. Intrinsics of the hand

  2. Wrist extensor

  3. Thumb extensor

  4. Thumb IP flexor

  5. Digital extensor

 

PREFERRED RESPONSE 4

 

This patient has sustained a displaced extension-type supracondylar fracture. The most commonly affected nerve in this setting is the anterior interosseous nerve (AIN). This will affect thumb IP flexion.

 

The most common neurapraxia after pediatric extension-type supracondylar fractures involve the AIN. It supplies the FPL (thumb IP flexion), the pronator quadratus, and the FDP of the index/long fingers. Subsequently, patients are often unable to make an “a-ok” sign. Most of these neuropraxias resolve without complication. The ulnar nerve is most commonly implicated with flexion-type supracondylar fractures.

 

Abzug et al. review management of supracondylar fractures. They note that the AIN

is most commonly injured nerve in extension type supracondylar fractures. They note that nerve injuries often resolve within 6-12 weeks.

 

Babal et al. completed a meta-analysis to determine the risk of neurapraxia associated with pediatric supracondylar fractures. The rate of traumatic neurapraxia was 11.4% amongst 5000 patients. The AIN was affected 34.1% of the time. AIN neurapraxia was most common in extension type injuries.

 

Figures A and B show an AP and lateral radiographs of a displaced pediatric supracondylar fracture

 

Incorrect Answers

Answer 1: The intrinsics are innervated by the ulnar nerve which is implicated in flexion-type supracondylar fractures Answer 2: The wrist extensors are innervated by the radial nerve. The radial nerve is not affected as commonly as the AIN Answers 3, 5: The digital extensors and the thumb extensors are innervated by the posterior interosseous nerve (PIN) which is not affected as commonly as the AIN

 

(OBQ15.143) Figure A is a pelvic radiograph of a healthy 20-month old girl with a limb leg discrepancy. A hip arthrogram under general anesthesia reveals concentric reduction of the left hip ONLY when the hip is positioned in 70 to 80 degrees of abduction. The next best step in treatment would be: Review Topic

 

 

 

  1. Observation

  2. Application of Pavlik harness

  3. Closed reduction and hip spica casting

  4. Open hip capsulorrhaphy +/- femoral osteotomy +/- pelvic osteotomy

  5. Open hip capsulorrhaphy, adductor and hamstring tenotomy, gastrocnemius recession +/- pelvic osteotomy

 

PREFERRED RESPONSE 4

 

Figure A shows left hip subluxation consistent with late presenting developmental hip dysplasia (DDH). The most appropriate treatment at this stage would be open hip

capsulorrhaphy, +/- femoral osteotomy +/- pelvic osteotomy

 

In children older than 18 months, open reduction is the preferred treatment for hip dysplasia. Open reduction is also considered when concentric closed reduction cannot be achieved or when excessive abduction (>60 degrees) is required to maintain reduction (due to increasing risk of avascular necrosis). The goal of open reduction is to remove the blocks to reduction and to increase stability by preforming soft-tissue or bone procedures.

 

Wenger et al. reviewed the surgical treatment of DDH. They say that the impediment to congruent reduction are the iliopsoas muscle, joint capsule, ligamentum teres, pulvinar, labrum, and transverse acetabular ligament. The most commonly used approaches are anterior and medial/anteromedial to the hip.

 

Gholve et al. reviewed DDH and the factors that contribute to secondary procedures. In their study, neither age nor unilateral vs bilateral dislocation had a direct influence on the outcome or the need for secondary procedures. The variable that significantly influenced the need for a secondary procedure was the concomitant use of a femoral osteotomy during initial surgical treatment. Of the 27 patients who did not have concurrent femoral osteotomy at index surgery, 19/27 (73%) required a secondary procedure (P<0.001).

 

Figure A shows an AP pelvic radiograph of an infant. Focusing on the left hip, there is lateral and superior subluxation of the femoral head, with the femoral head lateral to Perkin's line. The medial joint space is increased. The acetabular index is increased

>30 degrees. Shenton's line is disrupted. These are all features of DDH.

 

Incorrect Answers:

Answer 1: Observation would not be standard treatment of a persistently subluxed hip.

Answer 2: Pavlik harness is not indicated in older children Answer 3: Excessive abduction (>60 degrees) to maintain reduction has an increased risk of hip avascular necrosis. Answer 5: Adductor tenotomy, hamstring tenotomy, gastrocnemius recession would be required in patients with DDH due to underlying spasticity (e.g. cerebral palsy). This patient is otherwise healthy. In addition, patients who that do not have concurrent femoral osteotomy at index surgery often require secondary reconstructive procedures, due to recurrence.

 

(SAE09FA.100) A 23-year-old woman with a history of bilateral recurrent ankle sprains, progressive cavovarus feet, and a family history of high arches and foot deformities is seen for evaluation. Management consisting of bracing and physical therapy has been poorly tolerated. Heel varus is partially corrected with a Coleman block. There are thick calluses under the first metatarsal heads. Sensation to touch and

Weinstein monofilament is normal. Tibialis anterior and peroneus brevis are weak but present. What is the most appropriate management? Review Topic

 

  1. Continued bracing, physical therapy, and Botox injections in the triceps surae

  2. Peroneus longus to brevis transfer, medializing calcaneal osteotomy, and transfer of the extensor digitorum longus to the peroneus tertius

  3. Peroneus longus to brevis transfer, and transfer of the posterior tibial tendon to the tibialis anterior tendon

  4. Peroneus longus to brevis transfer, first metatarsal cuneiform dorsal closing wedge osteotomy, and lateralizing calcaneal osteotomy with proximal translation

  5. Triple arthrodesis

 

PREFERRED RESPONSE 4

 

The history and presentation are consistent with type I Charcot-Marie-Tooth (CMT), the most common form of hereditary peripheral motor sensory neuropathy. Type I CMT is the most common, occurring in 50% of patients with CMT, and is characterized by marked slowing of motor neuron velocities, and inconsistent slowing of sensory neuron velocities. Peroneus longus to brevis transfer is indicated to release the overpull of the peroneus longus, and restore the eversion and dorsiflexion function of the peroneus brevis. A lateralizing calcaneal osteotomy with proximal translation is indicated to correct heel varus given that the Coleman block only allows for partial correction of heel varus. Proximal translation of the posterior tuber corrects for the increased calcaneal dorsiflexion, improving the lever arm for the triceps surae. A medial column closing wedge osteotomy is often required to correct a rigid, or semirigid plantar flexed first ray to allow for a balanced, plantigrade foot. Triple arthrodesis is indicated for rigid, arthritic hindfoot deformities. Transfer of the posterior tibial tendon to the tibialis anterior is not indicated since it is an out-of-phase transfer. Transfer of the posterior tibial tendon, when performed, should be to the lateral aspect of the foot. A medializing calcaneal osteotomy would accentuate the heel varus. There is no indication for Botox in CMT; Botox injection of the calf would further weaken push-off during gait. Bracing of a progressive semirigid or rigid deformity is not recommended.

 

(SAE13PE.13) Figure 13 is the photograph of 18-month-old triplets with a lower-limb condition. What is the best initial treatment? Review Topic

 

 

  1. Vitamin D supplementation and serum level monitoring

  2. Mechanical axis alignment measurements on serial standing knee radiographs

  3. Genetic testing and counseling for short-stature syndrome

  4. Observation and clinical follow-up.

 

PREFERRED RESPONSE 4

 

These triplets exhibit genu varum and internal tibial torsion that can be part of normal development. Fetal packing is the likely major contributing cause for these triplets, however. Observation and follow-up will be sufficient. Bowing and torsion can be clinical features of vitamin D deficiency, Blount disease, and short-stature syndromes, but these are not the most likely diagnoses. Radiographic diagnosis of Blount disease may not be accurate at this age.

 

(SBQ13PE.56) A 22-month-old female is hospitalized with a fever and malaise. She is found to be bacteremic, and blood cultures grow methicillin-sensitive Staphylococcus aureus (MSSA). During her hospitalization, the pediatrician notices her arm is slightly swollen and appears painful to use. MRI is obtained and demonstrated in figure A. Which feature of the MRI suggests the need for surgical management? Review Topic

 

 

  1. Brodie's Abscess

  2. Osteomyelitis of the humeral metaphysis

  3. Subperiosteal abscess of the humerus

  4. Presence of subcutaneous air

  5. High risk of proximal humeral growth arrest

 

PREFERRED RESPONSE 3

 

This patient has osteomyelitis of the humerus with a large subperiosteal abscess. The presence of subperiosteal abscess necessitates surgical intervention for irrigation and debridement (I&D).

 

The diagnosis of acute osteomyelitis in children is made based upon a constellation of findings including pain, systemic signs of infection, elevated serum inflammatory markers, and imaging studies demonstrative of osseous infection. It is most commonly hematogenously spread to the metaphysis of immature bone, which is highly vascular prior to skeletal maturity. Treatment involves obtaining specimen for culture, empiric antibiotic treatment, and surgical irrigation and debridement of known abscesses. The end-point of treatment is return of pain-free functionality and the resolution of local/systemic signs of infection.

 

Conrad reviewed the management of acute hematogenous osteomyelitis and emphasized that surgical intervention in the presence of abscess can be both therapeutic and diagnostic: I&D can obtain culture and narrow the antibiotic plan.

 

Jones et. al. reviewed chronic pediatric osteomyelitis and report that surgery is the mainstay of treatment because removal of dead bone is essential for resolution of infection. This may be performed with sequestrectomy and curettage, with an emphasis on prevention of pathologic fracture, growth disturbances, bone loss, joint involvement, and permanent loss of function.

 

Figure A is a coronal STIR MRI image of the humerus demonstrating osteomyelitis

with extensive subperiosteal abscess.

 

Incorrect Answers:

  1. Brodie's abscess is a type of subacute osteomyelitis which remains indolent and creates a focal intra-osseous abscess. This is not demonstrated in the clinical image.

  2. The patient does have osteomyelitis of the humeral metaphysis, but this alone is not the indication for surgery. Uncomplicated osteomyelitis may be treated effectively with antibiotics alone.

  1. Presence of subcutaneous air is suggestive of necrotizing fasciitis, which is a surgical emergency. The MRI however does not demonstrate this finding.

  2. The infection and its surgical management both increase risk of injury to the proximal humeral physis and has the potential for growth arrest or angular deformity.

 

(SBQ13PE.3) Figure A demonstrates a physical examination maneuver in a 1 month old infant. What is this maneuver? Review Topic

 

 

 

  1. Ortolani Test

  2. Barlow Test

  3. Galeazzi Sign

  4. Patrick test

  5. Teratologic Sign

 

PREFERRED RESPONSE 1

 

Figure A shows a schematic image of the Ortalani test.

 

The Ortolani test, or Ortolani maneuver, is part of the physical examination for developmental dysplasia of the hip. It is used alongside the Barlow test to detect subluxated hips that are either reducible or irreducible. A positive sign is a distinctive 'clunk' which can be heard and felt as the femoral head relocates anteriorly into the acetabulum.

Guille et al. showed that the use of Pavlik harness has become the mainstay of initial treatment for the infant who has not yet begun to stand. If concentric reduction of the hip cannot be obtained, surgical reduction of the dislocated hip is the next step before they are able to stand.

Video V is a lecture discussing the hip examination of the infant. Incorrect Answers:

Answer 2: Barlow test is performed by adducting the hip while applying light

pressure on the knee, directing the force the femoral head posteriorly. A positive Barlow test would result in posterior subluxation/dislocation of the hip. Answer 3: Galeazzi test is performed by flexing an infant's knees in the supine position so that the ankles touch the buttocks. If the knees are not level then the test is positive, which indicates a potential congenital hip issue (e.g. DDH). Answer 4: Patrick or Faber test (flexion, abduction, external rotation) has been described both for stressing the SI joint and for isolating symptoms to the hip Answer 5: Teratologic dislocation of the hip is a term used to imply that the hip joint did not develop normally in utero, thus the hip is in a fixed dislocated position at birth.

 

(SAE07PE.17) An 11-year-old basketball player reports that he felt a painful pop in the left knee when he stumbled while running. He is unable to bear weight on the extremity and cannot actively extend the knee against gravity. Examination reveals a large knee effusion. A lateral radiograph is shown in Figure 7. Management should consist of Review Topic

 

 

 

  1. physical therapy for quadriceps strengthening exercises.

  2. a long leg cast with the knee fully extended.

  3. excision of the fragment.

  4. suture reattachment of the patellar tendon to the tibial tuberosity.

  5. open reduction and tension band fixation.

PREFERRED RESPONSE 5

 

The radiograph shows an avulsion fracture, or “sleeve fracture,” of the distal pole of the patella. The distal fragment is much larger than it appears on the radiograph because it largely consists of cartilage; therefore, excision of the fragment is contraindicated. The treatment of choice is open reduction and tension band fixation to correct patella alta and restore the extensor mechanism.

 

(SAE08OS.56) A 3-year-old boy has hip pain and refuses to walk. His right hip is tender with any range of motion. His temperature is 102.2 degrees F (39.0 degrees C). Laboratory studies reveal the following values: C-reactive protein level of 6.0, WBC count of 19,000/mm3 (normal 3,500 to 10,500/mm3), and an erythrocyte sedimentation rate of 45 mm/h (normal up to 20 mm/h). A hip sonogram demonstrates a large effusion in the right hip. What is the best treatment plan? Review Topic

 

  1. Hip aspiration

  2. MRI of the hip and pelvis

  3. Western Blot examination for Lyme disease

  4. Pediatric rheumatology consultation

  5. Admission for IV antibiotics

 

PREFERRED RESPONSE 1

 

The findings in this patient are classic for septic arthritis. If confirmed by aspiration, urgent debridement of the joint is required to help prevent long-term sequela of septic arthritis. IV antibiotics alone are not an appropriate treatment for septic arthritis of the hip. Additional prolonged work-up with rheumatology, Western Blot, and MRI would most likely delay the prompt surgical treatment of septic arthritis of the hip.

 

(SBQ13PE.61) Mutations in genes COL1A1/COL1A2 are responsible for the clinical manifestations of: Review Topic

 

  1. Marfan's syndrome

  2. Osteopetrosis

  3. Osteogenesis imperfecta

  4. Achondroplasia

  5. Multiple Epiphyseal Dysplasia

 

PREFERRED RESPONSE 3

 

Osteogenesis imperfecta is caused by mutations in type I collagen genes COL1A1 or COL1A2 in 85% of cases.

 

Pathological manifestions of osteogenesis imperfecta (OI) occur because of abnormal type 1collagen . Clinical manifestations include multiple fractures, blue sclera, and scoliosis. The genes responsible for normal collagen synthesis are COL1A1/COL1A2 and many mutations have been characterized which result in OI. Quantitative collagen defects result in a milder phenotype whereas qualitative collagen defects result in more severe clinical manifestations.

 

Zeitlin et al. reviewed osteogenesis imperfecta. They report that bone fragility in OI stems from decreased bone mass, disturbed organization of bone tissue, and altered bone geometry (size and shape). They found that cyclical intravenous pamidronate administration reduces bone pain and fracture incidence while increasing bone density and level of ambulation with minimal side effects.

 

Burnei et al. review the diagnosis and treatment of OI. They report that medical treatment with bisphosphonates, even in patients younger than age 2 years, have become widely accepted in the symptomatic treatment of OI. They report alendronate has proven to beneficially decrease fracture frequency while improving vertebral bone density and quality of life.

Illustration A shows the Sillence Classification of Osteogenesis Imperfecta. Incorrect Answers:

Answer 1: Marfan's syndrome is an autosomal dominant disorder that affects the Fibrillin-1 gene.

Answer 2: Osteopetrosis is an autosomal dominant disorder that affects the production of macrophage colony-stimulating factor. Answer 4: Achondroplasia is an autosomal dominant disorder that affects FGFR-3. Answer 5: Multiple Epiphyseal Dysplasia Type 1 and Type 2 are autosomal dominant diseases that affect cartilage oligomeric matrix protein and Type IX collagen, respectively.

 

(SAE07PE.29) A 2-year-old child is being evaluated for limb-length and girth discrepancy. As a newborn, the patient was large for gestational age and had hypoglycemia. Current examination shows enlargement of the entire right side of the body, including the right lower extremity and foot. The skin shows no abnormal markings, and the neurologic examination is normal. The spine appears normal.

Radiographs confirm a 2-cm discrepancy in the lengths of the lower extremities. Additional imaging studies should include Review Topic

 

  1. bone age of the left wrist.

  2. MRI of the spine.

  3. MRI of the brain.

  4. renal and abdominal ultrasonography.

  5. hip ultrasonography.

 

PREFERRED RESPONSE 4

 

The patient may have Beckwith-Wiedemann syndrome (BWS), which consists of exophthalmos, macroglossia, gigantism, visceromegaly, abdominal wall defects, and neonatal hypoglycemia. Hemihypertrophy develops in approximately 15% of patients with BWS. Patients with hemihypertrophy that is the result of BWS have a 40% chance of developing malignancies such as Wilms’ tumor or hepatoblastoma; therefore, frequent ultrasound screening is recommended until about age 7 years. The absence of nevi and vascular markings helps to rule out other causes of hemihypertrophy, such as neurofibromatosis, Proteus syndrome, and Klippel-Trenaunay syndrome. Bone age estimations are not accurate at this young age but may become more useful later to help predict the timing of epiphysiodesis procedures.

 

(SAE10PE.44) A 6-year-old boy is being treated for acute hematogeneous osteomyelitis of the distal femur with intravenous antibiotics. The best method to determine the success or failure of initial treatment is by serial evaluations of which of the following studies? Review Topic

 

  1. Radiographs

  2. MRI

  3. Erythrocyte sedimentation rate (ESR)

  4. CBC with differential

  5. C-reactive protein (CRP)

 

PREFERRED RESPONSE 5

 

Successful antibiotic treatment of acute osteomyelitis should lead to a rapid decline in the CRP. The CRP is the most sensitive study to follow the treatment of osteomyelitis. The CRP should decline after 48 to 72 hours of appropriate treatment. CBC and ESR are helpful in initial evaluation and diagnosis, but remain abnormal in the early phase of treatment regardless of response. Imaging studies are useful for surgical planning or secondarily if the CRP remains elevated.

 

(SAE13PE.111) The Coleman block test is used to test for Review Topic

 

  1. flexibility of the forefoot.

  2. flexibility of the hindfoot.

  3. valgus deformity of the hindfoot.

  4. tightness of tendo Achilles.

 

PREFERRED RESPONSE 2

 

The Coleman block test is used to determine the flexibility of the hindfoot. When a block is placed under the lateral border of the foot, the medial column is unsupported. As a result, the first metatarsal drops off the side of the block. If the subtalar joint is flexible, there is no fixed varus deformity of the hindfoot. The hindfoot will no longer be in varus from behind. The varus deformity of the hindfoot will be corrected. If there is no subtalar motion, the varus deformity remains fixed.

 

(OBQ14.193) A patient undergoes the procedure shown in Figure A. This patient is most likely to be Review Topic

 

  1. 45-year-old man with subtalar arthritis

  2. 8-year-old girl with congenital vertical talus

  3. 14-year-old male with talocalcaneal coalition involving less than 50% of middle facet and hindfoot valgus

  4. 30-year-old male with cavovarus foot and Charcot-Marie-Tooth disease

  5. 14-year-old male with equinovalgus foot and spastic diplegic cerebral palsy

 

PREFERRED RESPONSE 3

 

The procedure shown is subtalar arthroereisis. It is used as an adjunct spacer/distractor following tarsal coalition excision if hindfoot valgus remains following resection.

 

In the pediatric population, arthroereisis is one option to restore the alignment of the hindfoot after talocalcaneal coalition. Hindfoot deformity correction is required because resection of the coalition alone will often increase the hindfoot valgus

deformity. The arthroereisis implant prevents this valgus collapse. Another alternative to correct the hindfoot valgus deformity is a calcaneal lateral column lengthening osteotomy.

 

Khoshbin et al. reviewed the long-term outcomes of coalition resection in 24 patients (32 coalitions). Resected talocalcaneal (TC) coalitions had less inversion/eversion postoperatively than resected calcaneonavicular (CN) coalitions but there was no difference in outcome scores. They obtained favorable results when even resecting talocalcaneal coalition with >50% involvement of the middle facet and hindfoot valgus angles >16 °, which were considered historical contraindications to resection.

 

Zaw et al. reviewed tarsal coalitions. Radiographic signs of CN coalition include the anteater sign (elongated anterior calcaneal process), decreased CN gap, reverse anteater sign (elongated lateral navicular) and hypoplastic lateral talar head. Radiographic signs of TC coalition include obliterated middle facet on a Harris view (osseous coalition), irregular cortices/dysplastic sustentaculum tali on a Harris view (nonosseous), C-sign on a lateral view, talar beaking, short talar neck with concave inferior surface, narrow posterior facet, and non-visibility of the middle facet.

 

Giannini et al. reviewed subtalar arthroereisis with coalition resection in 14 feet in patients aged 9-18 years. They achieved 57% excellent, 21% good and 21% fair results. Regarding pain, 86% had improvement and 14% had no change. Regarding ROM, 93% had improvement, and 7% had no change. Better scores were seen in patients <14 years.

 

Figure A shows the implantation of an arthroereisis implant in the sinus tarsi. Illustration A comprises coronal CT images of talocalcaneal coalition.

 

Incorrect Answers:

Answer 1: Subtalar arthritis is a contraindication to arthroereisis. Other contraindications include active infection, or traumatic/surgical wound over the sinus tarsi.

Answers 2, 4, 5: Arthroereisis is not indicated for these conditions.

 

(SAE10PE.34) A 7-year-old girl with a known diagnosis of neurofibromatosis has neck pain and deformity. She has been wearing a soft cervical collar for the past 2 months with mild relief of her symptoms. An MRI scan shows several small neurofibromas on the left side of the cervical spine near the foramina at C6 and 7. A lateral cervical spine radiograph is shown in Figure 34. What is the most appropriate management? Review Topic

 

 

  1. Anterior and posterior spinal fusion

  2. Anterior spinal fusion

  3. In situ posterior fusion

  4. Halo traction correction and posterior fusion

  5. Continued soft cervical collar treatment

 

PREFERRED RESPONSE 1

 

With a diagnosis of neurofibromatosis and severe kyphosis, anterior and posterior treatment is needed to achieve correction and fusion. In situ fusion has a high failure rate with the kyphotic deformity and even with traction, correction of the kyphosis is not expected. Anterior treatment alone may achieve correction, but in neurofibromatosis only circumferential treatment has been shown to provide longterm stability.

 

(SAE09TR.21) A 7-year-old boy is seen in the emergency department with an isolated and displaced supracondylar humerus fracture and absent radial and ulnar pulses. Despite a moderately painful attempt at realignment, examination reveals that his hand remains pulseless. What is the next most appropriate step in management? Review Topic

 

  1. Order an urgent angiogram and then proceed to the OR

  2. Repeat the reduction in the emergency department and reassess.

  3. Perform open reduction through an anterior approach.

  4. Perform closed reduction and pinning in the OR and reassess the vascular status.

  5. Perform arterial repair and then stabilize the fracture.

 

PREFERRED RESPONSE 4

 

Displaced supracondylar humerus fractures in children may have associated vascular compromise. Decreased blood flow may be due to vessel injury, entrapment within the fracture site, kinking from fracture displacement, or from vessel spasm. Optimal initial treatment in the emergency department includes gentle realignment of the limb

and vascular assessment. Angiography is not required in isolated injuries as the level of the vessel compromise is always at the site of the fracture. When blood flow is not restored, the next best step in treatment is to proceed urgently to the operating room. A formal closed reduction and pinning is performed, and then the vascular status is reassessed. Exploration and vascular repair is required if the hand is cool, white, and without pulses.

 

(SAE09TR.42) A 9-year-old boy falls from a scooter and sustains the injury shown in the radiographs in Figure 26. After closed reduction and cast immobilization, what is the most likely complication that can result? Review Topic

 

 

 

  1. Growth arrest of the distal ulna

  2. Growth arrest of the distal radius

  3. Compartment syndrome

  4. Radioulnar synostosis

  5. Entrapment of the extensor pollicis longus (EPL) tendon

 

PREFERRED RESPONSE 1

 

The radiographs show a fracture of the distal radius and ulna physis. The most likely complication is growth arrest of the distal ulna. In contradistinction to physis fractures of the radius (growth arrest incidence of less than 5%), the incidence of growth arrest in the ulna is between 30% and 40%. Entrapment of the EPL tendon and cross union between the two bones is extremely rare.

(SAE07PE.87) A 7-year-old boy with spastic diplegia is a limited community ambulator. He has a moderately severe crouched gait. The parents request a treatment that will result in a permanent decrease in lower extremity muscle tone. This is best accomplished with Review Topic

 

  1. tone-reduction ankle-foot orthoses (AFOs).

  2. intramuscular injections of botulinum-A toxin.

  3. an intrathecal baclofen injection.

  4. selective posterior rhizotomy.

  5. fractional tendon lengthening of bilateral hamstring and gastrocnemius muscles.

 

PREFERRED RESPONSE 4

 

Posterior rhizotomy provides a permanent reduction in tone of spastic muscles. Potential drawbacks of the procedure include excessive muscle weakness, hip dislocation, and spinal deformity. Intramuscular botulinum-A toxin results in permanent blockade of presynaptic release of acetylcholine across the neuromuscular junction. The clinical effect usually resolves after 3 to 6 months due to neural regeneration. Tone-reduction AFOs have not been shown to reduce tone. A baclofen pump could offer prolonged reduction in tone, but not a single intrathecal injection.

 

(SAE08OS.168) A 10-year-old girl sustained a Gartland type III supracondylar fracture after falling off a trampoline 1 hour ago. She has a well perfused hand but no palpable pulses. The remainder of her examination is otherwise normal. What is the next step in treatment? Review Topic

 

  1. Oblique radiographs

  2. Arteriogram

  3. Exploration of the brachial artery at the elbow

  4. Closed reduction and pinning of the elbow

  5. Open reduction and internal fixation of the elbow

 

PREFERRED RESPONSE 4

 

Most supracondylar fractures are extension type and a Gartland type III is defined as a fracture that is completely displaced (ie, no posterior cortical hinge). Treatment consists of closed reduction and pinning. Controversy exists over the use of a medial and lateral pin versus multiple lateral pins. Although uncommonly needed, the surgeon should be prepared to open the fracture to achieve adequate reduction. A careful preoperative examination should be performed and documented. There is a growing body of evidence that patients with a normal neurovascular examination can be treated in a delayed fashion; nevertheless, the initial evaluation cannot be delayed.

If there is evidence of vascular compromise, the fracture should be reduced and pinned urgently and the limb reevaluated.

 

(OBQ15.2) A 6 year-old-boy falls from his bunk bed and suffers the injury seen in Figures A and B. Upon presentation to the emergency room he is noted to have a pink hand with brisk capillary refill, but no palpable pulses. After closed reduction in the operating room and the procedure seen in Figures C and D, he remains well perfused, pulses are still not palpable, but triphasic pulses can be heard on doppler examination. What is the most appropriate course of action? Review Topic

 

 

 

  1. Remove fixation and repeat vascular exam

  2. Add a medial pin, remove two of the lateral pins and repeat vascular exam

  3. Maintain hardware position, apply a soft dressing and observe

  4. Maintain hardware position, apply long arm immobilization and observe

  5. Surgically explore the antecubital fossa

PREFERRED RESPONSE 4

 

A "pink pulseless" hand, especially those with strong triphasic doppler signals can be treated with observation. In the presented case it would be appropriate to place long arm immobilization (cast or splint) and observe.

 

Pediatric supracondylar humerus fractures presenting with vascular compromise can present a challenge for clinical decision making. If the hand lacks adequate perfusion on presentation it requires emergent treatment with closed reduction and pinning. If perfusion does not return, then surgical exploration of the antecubital fossa is required. Treatment of the "pink pulseless hand" remains controversial. If the hand is well perfused but lacks palpable pulses then it should be expeditiously reduced and pinned. If the hand remains well perfused with triphasic doppler signals, no additional intervention is required. Recommendations vary if the hand is perfused, and pulses cannot be detected on palpation or doppler.

 

If there is a normal neurovascular exam on presentation, but abnormal after reduction and pinning, then one must consider that the reduction may have entrapped the brachial artery. If hardware removal and fracture re-displacement does not improve pulses/perfusion of the hand, or if the fracture can not be reduced without diminishing perfusion, then open reduction is indicated to evaluate the brachial artery.

 

Shah et al. present their preferred algorithm for treatment of a pink pulseless hand. They note that even with good clinical perfusion there may be a vascular injury or entrapment of the brachial artery. Thus they recommend if the hand is pink with no palpable pulse, perform a doppler exam. Triphasic doppler signals allow for observation, while abnormal or non-detectable doppler signals require surgical exploration. If an AIN or median nerve injury is present, exploration is considered. A diagram of their algorithm can be seen in Illustration A.

 

Weller et al. present a retrospective review of 1297 supracondylar humerus fractures. They had 20 patients (1.5%) with a pink pulseless hand with good doppler signals following closed reduction and fixation. These patients were treated with close observation. One patient developed inadequate perfusion nine hours after reduction and required vascular repair. The remaining 19 patients regained palpable pulses prior to discharge or prior to the first follow-up visit. They recommend observation despite non-palpable pulses as long as other signs of perfusion (capillary refill and doppler signals) suggest the hand is well perfused.

 

Figures A and B are AP and lateral radiographs of a type III supracondylar humerus fracture. Figures B and C demonstrate this fracture following closed reduction and percutaneous pinning with three lateral pins. Illustration A is a flow chart of a proposed decision making algorithm for a pink pulseless hand as presented by Shah et al.

 

Incorrect Answers:

Answer 1- Removal of fixation and re-displacement of the fracture would be indicated if a previously normal neurovascular exam became abnormal following closed

reduction and fixation. Answer 2- Changing the pattern of fixation would only be indicated if the current construct did not provide stable fixation of the fracture. Answer 3- Following reduction and fixation of a supracondylar humerus fracture the most appropriate dressing is rigid long arm immobilization in a splint or cast. If there is concern for edema or perfusion then a cast should be bivalved. Answer 5- Surgical exploration of the antecubital fossa would be indicated if the hand had inadequate perfusion, or if the fracture could not be reduced and fixed without causing a notable decrease in pulses or perfusion.

 

(SAE08OS.160) A 10-month-old infant has a deformity of the right foot. Radiographs, including simulated weight-bearing AP and lateral views and a maximum plantar flexion lateral view, are shown in Figures 57a through 57c. Initial management of the foot should consist of Review Topic

 

 

 

  1. manipulation and cast application.

  2. basic orthopaedic shoes with a scaphoid pad and reinforced medial counter.

  3. open reduction of the talonavicular joint and Achilles tendon lengthening.

  4. lateral column lengthening osteotomy.

  5. observation.

PREFERRED RESPONSE 1

 

The radiographs show a congenital vertical talus. This is confirmed on the maximum plantar flexion lateral view which shows failure of the long axis of the first metatarsal to align with the long axis of the talus. This finding is caused by a fixed dorsal dislocation of the navicular on the head of the talus. The initial treatment should consist of manipulation and serial cast application in an attempt to elongate the contracted dorsolateral tendons, joint capsules, and skin. Surgery is always required to complete the correction. Traditionally, surgical treatment consisted of lengthening of the dorsolateral tendons, release of the talonavicular joint capsule, and lengthening of the Achilles tendon. Recently, Dobbs and associates reported the successful use of manipulation and cast immobilization, followed by pinning of the talonavicular joint and percutaneous tenotomy of the Achilles tendon in patients with idiopathic congenital vertical talus. There are no studies documenting the effectiveness of orthoses for the treatment of this condition. Lateral column lengthening may be indicated in older individuals with a symptomatic flexible flatfoot, especially those with neurologic conditions. Observation may be indicated in a young child with a painless flexible flatfoot.

 

(SAE10PE.87) A 12-year-old boy has had left thigh pain for the past 4 months. Examination shows lack of internal rotation and abduction, and external rotation with hip flexion. A radiograph is shown in Figure 87. What is the most appropriate treatment? Review Topic

 

 

 

  1. Physical therapy

  2. In situ pinning

  3. Reduction and percutaneous pinning

  4. Surgical dislocation of the hip with reduction under direct vision

  5. Spica casting

 

PREFERRED RESPONSE 2

The patient has a stable slipped capital femoral epiphysis (SCFE). Preferred treatment of stable SCFE is in situ pinning. In situ fixation of stable SCFE has an extremely low rate of osteonecrosis. Gentle postural reduction with hip capsulotomy or surgical dislocation of the hip with reduction has been advocated for unstable SCFE.

 

(SAE08OS.76) Which of the following foot deformities is commonly seen in patients with Charcot-Marie-Tooth disease? Review Topic

 

  1. Plantar flexed first metatarsal and hindfoot valgus

  2. Plantar flexed first metatarsal and hindfoot varus

  3. Dorsiflexed first metatarsal and hindfoot valgus

  4. Dorsiflexed first metatarsal and hindfoot varus

  5. Abducted first ray and hindfoot valgus

 

PREFERRED RESPONSE 2

 

Charcot-Marie-Tooth disease is an inherited progressive motor and sensory peripheral neuropathy. Patients commonly present with bilateral pes cavovarus deformities characterized by plantar flexed metatarsals and a varus hindfoot.

 

(OBQ13.151) A 14-year-old male sustains the injuries shown in Figures A and B after a fall off the roof of his house. What is the most appropriate management? Review Topic

 

 

 

  1. Hip spica in abduction

  2. Dynamic hip screw with trochanteric side plate

  3. Cephalomedullary nail

  4. Physis-sparing cancellous screws

  5. Transphyseal cancellous screws

 

PREFERRED RESPONSE 5

 

The radiographs show a transcervical (Delbet II) femoral neck fracture. Transphyseal cancellous screws are indicated for fixation of the femoral neck fracture in this case.

 

Pediatric femoral neck fractures are classified into transphyseal, transcervical, cervicotrochanteric and intertrochanteric (Delbets I-IV) respectively. Fixation type varies by author, but generally speaking, Types I-III can be treated with smooth wires for patients 0-3yrs of age, 4.5-6.5 mm cannulated screws 4-10yrs of age, or l 6.5-

7.3mm cannulated screws inpatients >10 yrs old. Type IV fractures are treated with pediatric or adult DHS depending on age.

 

Boardman et al. reviewed hip fractures in children. They found that type II and III fractures are displaced at presentation, and even reduced fractures treated with cast immobilization may later displace. They recommend anatomic reduction and surgical stabilization for displaced fractures. Transphyseal fixation is preferred over physis-sparing fixation to achieve stability except for children aged <4-6 years where physis-sparing fixation is preferred, given the shorter healing period.

 

Figures A and B are AP and lateral radiographs of the right hip showing a transcervical femoral neck fracture in a skeletally immature individual. Illustration A shows transphyseal cancellous screw fixation of the same fracture. Illustration B is a table depicting recommended treatment for different fracture types. Illustration C depicts physis-sparing fixation.

 

Answer 1: Hip spica treatment is only appropriate for patients <6 years old with Types III and IV fractures. Answer 2: DHS or pediatric DHS is indicated for Type IV fractures. Answer 3: Cephalomedullary nail fixation is not indicated for pediatric femoral neck fractures.

Answer 4: Physis-sparing screws may be performed for Type III fractures. In Type II fractures, there will not be enough screw purchase in the femoral head fragment. For Type II fractures, transphyseal fixation provides the greatest stability.

 

(OBQ13.199) Which of the following is most commonly inherited as a X-linked recessive disorder? Review Topic

 

  1. Charcot-Marie-Tooth disease

  2. Marfan's syndrome

  3. Larsen's syndrome

  4. Duchenne's muscular dystrophy

  5. Turner's syndrome

 

PREFERRED RESPONSE 4

 

Duchenne's muscular dystrophy is an X-linked recessive disorder.

 

Duchenne's muscular dystrophy is caused by a mutation in the dystrophin gene. X-linked recessive inheritance is a mode of inheritance in which a mutation in a gene on the X chromosome causes the phenotype to be expressed (1) in males (who are necessarily hemizygous for the gene mutation because they have only one X chromosome) and (2) in females who are homozygous for the gene mutation (i.e., they have a copy of the gene mutation on each of their two X chromosomes).

 

Zane reviewed paediatric neuromuscular disorders. Duchenne's muscular dystrophy patients will show markedly elevated CPK levels (10-200x normal). Muscle biopsy will show connective tissue infiltration, foci of necrosis and absent dystrophin with staining.

 

Illustration A shows an illustration of Gowers sign. This indicates weakness of the proximal muscles. It is characteristic of patients with Duchenne's muscular dystrophy.

 

Incorrect Answers:

Answer 1: Charcot-Marie-Tooth disease is most commonly inherited as an autosomal dominant disorder. Rarely is it X-linked recessive. Answer 2: Marfan's syndrome is most commonly inherited as an autosomal dominant disorder.

Answer 3: Larsen's syndrome may be autosomal dominant (AD) or recessive (AR). Answer 5: Turner's syndrome is a 45 XO disorder.

 

(SAE13PE.95) An 11-year-old boy with bipolar disorder fell from a tree and sustained an open fracture dislocation of the right ankle with extensive abrasions of the leg. Immediate irrigation, debridement, reduction, and provisional fixation with Kirschner wires was performed. Twenty-four hours later, the patient’s blood pressure is 190/100 mm Hg and pulse rate is 120. He has required only 1 dose of an oral analgesic for pain control. His foot and ankle are markedly swollen, but there is no pain on passive extension of the toes. The dorsalis pedis pulse cannot be palpated. What is the most appropriate next treatment step? Review Topic

 

  1. Remove the Kirschner wires and reposition the ankle

  2. Measure foot compartment pressures

  3. Order an echocardiogram

  4. Order serum blood urea nitrogen and creatinine levels

PREFERRED RESPONSE 2

 

The most common symptom of compartment syndrome in the extremities is intense pain. Compartment syndrome can be difficult to diagnose in children and patients who are comatose, nonverbal, and/or mentally compromised because they may not be able to properly express their level of pain. In compartment syndrome of the leg, pain on passive extension of the toes is the most frequent clinical diagnostic finding. However, in compartment syndrome of the foot, pain on passive extension of the toes may or may not be present. Swelling and absence of the dorsalis pedis pulse may be expected findings with extensive trauma to the foot, making the clinical diagnosis even more difficult. Repositioning the ankle will add to further swelling. The clinician must be alerted regarding elevations in blood pressure and pulse because such elevations may be the only manifestation of the deeper problem. The transient blood pressure elevation does not require cardiac screening with electrocardiogram or echocardiogram as in chronic hypertension. Kidney function testing is not necessary because the blood pressure elevation is not renal in origin. Compartment pressures should be measured immediately in the foot and will require anesthesia in the pediatric age group.

 

(SBQ13PE.91) An 12-year-old girl presents with right hip pain. Bilateral frog laterals are shown in Figure A. Laboratory work-up reveals TSH 11 mIU/L (Ref range: 0.4-4.0 mIU/L) and Free T4 is 0.5 ng/dL (Ref range: 0.7-1.9 ng/dL). What is the most appropriate treatment recommendation? Review Topic

 

 

 

  1. In situ pinning of right hip

  2. Protected weight bearing and MRI of right hip

  3. Immediate endocrine referral and treatment

  4. Open biopsy right hip

  5. In situ pinning bilateral hips PREFERRED RESPONSE 5

In patients with slipped capital femoral epiphysis (SCFE) and concomitant endocrinopathy, bilateral pinning is the recommended treatment.

 

This patient's laboratory values reveal hypothyroidism, which increases the risk of bilateral involvement. Thus, the most appropriate treatment recommendation is surgical fixation of both hips.

 

Wells et al. analyzed 131 SCFEs over a 30-year period. The authors noted that 100% of patients with associated endocrinopathy went onto contralateral slip and recommended not only prophylactic pinning, but in those with open triradiate cartilage, recommended preventative screening with TSH/Free T4 laboratory studies.

 

Riad et al. followed 90 patients and analyzed impact of age, gender and race on contralateral slip development. Girls under the age of 10 and boys under the age of 12 had a significantly increased risk of contralateral involvement. Therefore, the authors recommended contralateral pinning for girls and boys that met those age criteria, respectively.

Figure A exhibits a right SCFE on bilateral frog lateral views. Incorrect Answers:

Answer 1: Due to the presence of hypothyroidism, bilateral pinning should be recommended due to the risk of contralateral slip Answer 2: MRI not needed because radiographs have provided the diagnosis. Answer 3: Referral can occur following surgery, but the most appropriate next treatment is surgery. Answer 4: Biopsy has no role in this case; no lesions noted in radiographs.

 

(SAE11OS.33) Radiographs of a 7-year-old child show mid-diaphyseal fractures of the radius and ulna. Closed reduction with sedation in the emergency department is performed. Postreduction radiographs demonstrate 18 degrees angulation, 30% translation, and what appears to be 20 degrees of rotational malalignment. Based on these findings, what is the next most appropriate step in management? Review Topic

 

  1. Another attempt at closed reduction in the operating room

  2. Open reduction with plating of the radius only

  3. Open reduction with plating of the ulna only

  4. Open reduction with plating of both the radius and ulna

  5. Close monitoring with follow-up radiographs in 1 week

 

PREFERRED RESPONSE 5

In children younger than 8 years of age, acceptable reduction parameters for fractures of the forearm are less than 20 degrees of angulation, 100% translation, and less than 45 degrees of malrotation. Weekly monitoring for loss of reduction and unstable fractures requiring further intervention is needed. When acceptable alignment can be maintained, good outcomes can be expected in this age group. In patients older than 10 years, angulation of less than 10 degrees, full translation, and malrotation of 30 degrees can be accepted. When surgical treatment is indicated, plating of one or both bones is acceptable. However, in this patient, the reduction is acceptable so a repeat closed reduction attempt and surgical treatment are not needed.

 

(SBQ13PE.17) A 14 year-old girl falls from the monkey bars, sustaining the injury shown in Figures A and B. This is a closed injury and she is neurovascularly intact. When deciding on optimal treatment, what are the acceptable parameters for angulation, malrotation, and bayonet apposition, respectively, in this patient? Review Topic

 

 

 

  1. 10 degrees, 30 degrees, Yes, allowed

  2. 10 degrees, 0 degrees, Yes, allowed

  3. 0 degrees, 0 degrees, None

  4. 0 degrees, 10 degress, None

  5. 10 degrees, 15 degrees, None

 

PREFERRED RESPONSE 3

 

Both bone fractures that occur in adolescents, especially in patients close to skeletal maturity, have very little potential for remodeling, and therefore, require anatomic restoration of the deformity.

 

As patients get older and closer to skeletal maturity, closed reduction alone typically is not sufficient to obtain and maintain anatomic reduction. Often, operative intervention is required via either intramedullary nailing (IMN) or plating.

Hertel et al. retrospectively reviewed diaphyseal plating of over 160 both bone forearm fractures in a wide age range of patients, which included adolescent patients (16 years old). The authors reported over a 96% union rate, low overall complication rate, and near zero refracture rate following subsequent hardware removal. The authors determined internal fixation with 3.5mm low contact dynamic compression plates sufficient for optimal results.

 

Baldwin et al, in this systematic review and meta-analysis compared the use of IMN and plate osteosynthesis for the treatment of pediatric both bone fractures. A total of 12 studies were included for analysis. The authors noted excellent union and outcome results for both treatment modalities, although although there was a significantly higher rate of hardware removal (94% vs. 49%) in IMNs. While there was a slight trend toward increased rates of non-union with IMN, it was not statistically significant.

 

Figures A and B show AP and lateral radiographs of a both bones forearm fracture in a near skeletally mature individual. Despite minimal angulation on the lateral, the deformity present in the coronal plane is unacceptable. Illustration A depicts the recommended parameters suggested by Baldwin et al. JOT 2014 for angulation, malrotation and bayonet apposition.

 

Incorrect answers:

Answer 1: These paramaters may be acceptable for someone under the age of 10, but are unacceptable for a 14-year-old patient. Answer 2: These parameters are unacceptable for a 14-year-old. Bayonet apposition is unacceptable over the age of 10.

Answer 4: While these parameters are very close to anatomic, for patients near skeletal maturity, even 10 degrees malrotation is unacceptable. Answer 5: Anatomic restoration is desired, and even minimal deformity is unacceptable in the case presented here

 

(SAE08OS.71) In a child with acute hematogenous osteomyelitis (AHO), what test is most likely to be abnormal? Review Topic

 

  1. WBC count

  2. Blood culture

  3. Bone culture

  4. Thyroid function test

  5. C-reactive protein

 

PREFERRED RESPONSE 5

Acute hematogenous osteomyelitis (AHO) affects 1 in 5,000 children and historically (prior to antibiotics) nearly half died due to the disease. Fortunately, the mortality rate is much lower today (less than 1%), but prompt diagnosis and treatment is critical to decrease the morbidity of this disease. Often many of the laboratory values may not be elevated, and cultures may not be positive; in fact, the WBC count and cultures (even bone) are positive less than half the time. The erythrocyte sedimentation rate is positive over 90% of the time, and often highly elevated but does not peak for 3 to 5 days. The C-reactive protein is elevated in 98% of patients with AHO, and decreases quickly after appropriate antibiotic therapy. The differential diagnosis of AHO is long, and includes bony tumors and leukemia, which should be considered, especially if the child fails to respond to appropriate treatment.

 

(SAE10PE.58) An 8-year-old boy weighing 70 lb sustains a displaced diaphyseal femur fracture and is treated with two flexible retrograde intramedullary rods. What is the most common complication following treatment with this technique? Review Topic

 

  1. Limb-length discrepancy

  2. Mechanical irritation around the knee

  3. Quadriceps weakness

  4. Malunion

  5. Patellofemoral pain

 

PREFERRED RESPONSE 2

 

Flexible retrograde intramedullary nailing is now the preferred treatment for most length-stable diaphyseal femur fractures in school-aged children. The most commonly described complication is irritation about the knee at the rod insertion sites that resolves with rod removal. Limb-length discrepancy and weakness have also been described at lower rates. Malunion or rod bending is usually related to placement of the rods in an unstable fracture pattern or in a larger patient.

 

(SAE07PE.13) Which of the following findings is most prognostic for the ability of a young child with cerebral palsy to walk? Review Topic

 

  1. Ability to sit independently by age 2 years

  2. Ability to creep by age 2 years

  3. Ability to roll by age 2 years

  4. Pattern of cerebral palsy (quadriplegia, diplegia, hemiplegia)

  5. Type of motor dysfunction (spastic, ataxic, dyskinetic, hypotonic)

 

PREFERRED RESPONSE 1

 

Several studies have shown that sitting ability by age 2 years is highly prognostic of walking. Molnar and Gordon reported that children not sitting independently by age 2 years had a poor prognosis for walking. Wu and associates reported that children sitting without support by age 2 years had an odds ratio of 26:1 of walking compared with those unable to sit. This was far higher than the odds ratios for cerebral palsy location, motor dysfunction, crawling, creeping, scooting, or rolling.

 

(SBQ13PE.27) A 15-year-old male patient presents requesting clearance to perform in the Special Olympics. He has had abnormal facies, has had mental developmental delay and cytogenetic analysis confirmed abnormalities on chromosome 21. Orthopaedically, he has been treated in the past for an elbow dislocation as well as bilateral patellar dislocation. He has already visited his cardiologist and endocrinologist and has been cleared. He has no complaints, denies any pain, difficulties with walking and reports that his training has been going well. What is the next best step? Review Topic

 

  1. With a normal physical exam, patient can be cleared for participation

  2. MRI bilateral knees and elbow to ensure no ligamentous injury

  3. Referral to a neurologist for clearance

  4. Lateral cervical spine flexion and extension radiographs

  5. AP pelvis radiograph

 

PREFERRED RESPONSE 4

 

Ruling-out C1-C2 instability with flexion/extension radiographs is necessary prior to any spine surgery or participation in sports in patients with Down's Syndrome.

 

Patients with Down's Syndrome typically present with generalized ligamentous laxity and decreased tone. Thus, dislocations (elbow or patella) along with asymptomatic instability in the cervical spine can commonly occur. Imaging analysis is necessary prior to sports participation.

 

McKay et al. performed a systematic review to summarize all congenital causes of cervical instability. They found in Down's patients, cervical instability due to ligamentous laxity is found mostly at C1-2. They recommend asymptomatic patients

with an ADI <4.5mm can resume unrestricted activities, while asymptomatic patients between 4.5-10mm should not participate in contact sports. With ADI >10mm OR symptoms/cord changes on MRI, surgery is recommended. Symptomatic patients with ADI between 4.5-10mm should be observed with activity restriction.

 

Dedlow et al. outlines the most recent 2011 update and guidelines for cervical instability in Down's syndrome patients. One of the major highlighted changes is the emphasis placed on radiographic re-examination, regardless of prior clearance. Re-examination should occur prior to participation in sports and/or the onset of new symptoms.

 

Illustrations A, B and C highlight the C1-2 instability on flexion-extension lateral radiographs. Careful attention can be placed on the relationship of the anterior arch of C1 and the dens (Illustration C). This allows for the measurements of ADI and the space available for the cord (SAC), which is highlighted in Illustration D.

 

Incorrect answers:

Answer 1: Even if the patient was previously cleared, he requires re-examination prior to clearance to participate in sports. Answers 2, 3: While prior injuries/treatments along with a neurologic examination may come into play if symptoms exists, cervical spine examination remains a priority. Answer 5: An AP pelvis radiograph is not warranted here (unless symptoms exists); still the cervical spine remains the priority/emphasis for clearance.

 

(SAE10PE.67) A 5-year-old boy reports intermittent left elbow pain. History reveals that he injured his elbow 4 months ago, but had no treatment. He is now using his arm normally but reports pain almost daily. Examination reveals tenderness over the lateral epicondyle and a prominence is evident. Range of motion is from -5 degrees to

120 degrees. Radiographs are shown in Figure 67. Management should include Review Topic

 

 

 

  1. open reduction and internal fixation.

  2. cast immobilization.

  3. percutaneous pin fixation.

  4. observation, with follow-up in 3 months.

  5. an MRI scan of the elbow.

 

PREFERRED RESPONSE 1

 

The patient has a nonunion of the lateral condyle of the left humerus. Observation or cast treatment at this stage is not likely to lead to healing of the fracture. MRI will not add any additional information. Open reduction, with minimal posterior soft-tissue stripping, is recommended to establish union of the fracture. Local or other bone graft may also be required. There are no studies that indicate that the displaced fracture will heal with late percutaneous fixation.

 

(SBQ13PE.40) Immediate spica casting is most appropriate for which of the following?: Review Topic

 

  1. 2-month-old girl with a displaced spiral mid-diaphyseal femur fracture

  2. A 26-month-old boy with a displaced spiral mid-diaphyseal femur fracture with <2 centimeters of shortening

  3. 3-day-old with teratologic right hip dislocation

  4. 9-year-old boy with a displaced spiral mid-diaphyseal femur fracture

  5. 12-year-old girl weighing 90 pounds with a displaced spiral mid-diaphyseal femur fracture

 

PREFERRED RESPONSE 2

 

Children older than 6 months and up to 6 years with diaphyseal femur fractures may be treated with spica casting.

 

Children younger than 6 months may only require a Pavlik harness. Immediate spica casting is indicated in children 6 months to 6 years with less than 2 cm of shortening. Each additional cm of shortening at presentation doubles the risk of loss of reduction (1 cm: 12%, 2 cm: 24%, 3 cm; 50%).

 

Kocher et al. provide the AAOS Clinical Practice Guideline for the treatment of pediatric diaphyseal femur fractures. Their recommendation for diaphyseal femur fractures with <2 cm shortening in children 6 months to 5 years is Grade B, based on Level II evidence (two level II and one level I study). They recommend early spica or traction with delayed spica. They suggest early spica, as this is more convenient than traction.

 

In an earlier article, Flynn et al. review the management of pediatric femoral shaft fractures. They recommend early spica casting for the child between 1 and 6 years in

low-energy femoral fractures with up to 2 cm shortening. Illustration A shows several styles of lower extremity spica casts.

Incorrect answers:

Answer 1: Children less than 6 months may be treated with spica casting but Pavlik harness is generally favored due to its ease of application and use and lower risk of skin complications

Answer 3: Spica casting plays a role in the management of DDH but teratologic dislocations are rarely reducible without open reduction. Answers 4 and 5: Spica casting will not adequately control femoral fractures in older children and is poorly tolerated in these age groups.

 

(SAE08OS.173) An 11-year-old boy reports the acute onset of elbow pain and swelling after pushing his brother. The patient's mother and a younger sibling have experienced numerous fractures. You note that the patient and his mother have blue sclera and normal-appearing teeth. A radiograph of the elbow is shown in Figure 60. This patient's disorder is most likely the result of Review Topic

 

 

 

  1. parental abuse.

  2. abnormal osteoclast function.

  3. defective N-Ac-Gal-6 sulfate sulfatase enzyme.

  4. a quantitative defect of type I collagen synthesis.

  5. vitamin D deficiency.

 

PREFERRED RESPONSE 4

 

Osteogenesis imperfecta (OI) is a genetically determined disorder of type I collagen synthesis characterized by bone fragility. This patient sustained a displaced fracture of the olecranon apophysis after relatively minor trauma. Physical examination reveals distinctly blue sclera. His mother and younger sibling have experienced numerous fractures suggesting a family history of bone fragility. The patient's history, clinical features, and family history are consistent with a diagnosis of Sillence type I-A OI. Type I OI is the mildest and most common form. Inheritance is autosomal dominant.

Type I is subclassified into the A type (absence of dentinogenesis imperfecta) and B type (presence of dentinogenesis imperfecta). These individuals have blue sclerae, and although the initial fracture usually occurs in the preschool years, it may occur at any age. Furthermore, olecranon apophyseal fractures that occur after relatively minor trauma have been associated with type I OI. Cells from individuals with type I OI largely demonstrate a quantitative defect of type I collagen; they synthesize and secrete about half the normal amount of type I procollagen. In this patient, there are no indications that the child has been subjected to abuse. Radiographs of the elbow show no evidence of osteopetrosis (due to abnormal osteoclast function) or rickets (due to a deficiency of vitamin D). Morquio syndrome (characterized by a defect of the enzyme N-Ac-Gal-6 sulfate sulfatase) is not associated with blue sclera.

 

(SAE13PE.106) Which gene correlates with severity of disease in spinal muscular atrophy (SMA)? Review Topic

 

  1. Peripheral myelin protein 22 (PMP22)

  2. Survival motor neuron I (SMN1)

  3. Survival motor neuron II (SMN2)

  4. Dystrophin

 

PREFERRED RESPONSE 3

 

SMA is caused by a deficiency in SMN protein. Deficiency of SMN protein leads to progressive loss of anterior horn cells and progressive muscle weakness. The severity of disease is directly related to the amount of reduction in circulating levels of SMN proteins, which are encoded by 2 alleles of the SMN1 gene and multiple copies of the SMN2 genes on chromosome 5. Affected patients with all types of SMA will have functional loss of both SMN1 genes, so this does not differentiate disease severity. Disease severity depends on the number of functional copies of SMN2 that remain. Patients with SMA1 have only 1 functioning SMN2 gene, whereas the milder forms, SMA types 2 and 3, have multiple copies that produce higher levels of SMN protein. The other choices are not associated with spinal muscular atrophy. Defects in PMP22 are the cause of 70% to 80% of cases of Charcot-Marie-Tooth disease. Mutations in the dystrophin gene cause Duchenne muscular dystrophy.

(SAE13PE.114) Figures 114a and 114b are the radiographs of a 5-year-old girl who fell from monkey bars. What is the first consideration regarding treatment for this injury? Review Topic

 

 

 

  1. Open reduction of the radiocapitellar joint

  2. Reconstruction of the annular ligament

  3. Closed reduction of the radial head dislocation and placement of a pin across the radiocapitellar joint

  4. Accomplishing and maintaining a reduction of the ulnar fracture

 

PREFERRED RESPONSE 4

 

The radiographs show an ulnar shaft fracture with a radial head dislocation (Monteggia fracture). The ulnar shaft fracture requires reduction, either closed or open. Once the ulnar fracture has been reduced, usually the radiocapitallar joint will relocate with closed manipulation. In patients for whom the radiocapitallar joint does not relocate after ulnar reduction, or in patients with missed Monteggia injury, reduction of the radiocapitellar joint usually requires opening of the elbow capsule and reconstruction of the annular ligament.

 

(OBQ13.163) A 7-year-old sustains the isolated injury shown in Figures A and B. On physical examination there is no evidence of soft tissue compromise and he is able to make an okay sign, give a thumbs up sign and cross his fingers. Which treatment will minimize complications? Review Topic

 

 

  1. Observation alone

  2. Closed reduction with casting in > 90 degrees of flexion

  3. Closed reduction with casting at 90 degrees of flexion

  4. Closed reduction and a percutaneous pinning construct using laterally based pins

  5. Closed reduction and a percutaneous pinning construct using crossed pins

 

PREFERRED RESPONSE 4

 

This patient has sustained an extension type supracondylar fracture (Gartland Type 3). The optimal treatment is closed reduction and a percutaneous pinning construct using laterally based pins.

 

Supracondylar fractures are common pediatric elbow injuries. Extension type injuries account for 95-98% of all cases. Non-displaced injuries may be treated conservatively. Displaced fractures are treated with closed reduction and percutaneous pinning. Use of laterally divergent pin constructs avoids risk of ulnar nerve injury, while maintaining satisfactory fracture alignment.

 

Slobogean et al. retrospectively reviewed pediatric supracondylar fractures to identify if there was an increased risk of ulnar nerve injury with crossed pin configurations. They found a higher incidence of ulnar nerve injury with crossed pinning constructs. For every 28 crossed pin constructs, there was one ulnar nerve injury identified.

 

Woratanarat et al. retrospectively reviewed supracondylar fractures to identify differences in the outcomes between lateral pinning versus crossed pinning constructs. No difference was found for loss of fixation or development of late deformity between the two groups. Crossed pin constructs were associated with a 4.3 times higher risk of iatrogenic ulnar nerve injury.

 

Omid et al. review pediatric supracondaylar humerus fractures. They note that lateral pinning is as stable as crossed pinning when appropriately spread-out at the fracture line. This also avoids the risk of ulnar nerve injury.

 

Figures A and B show AP and lateral radiographs of a pediatric supracondylar fracture. This would be classified as a Gartland Type 3.

 

Incorrect Answers:

Answer 1: Observation is not indicated in this clinical scenario.

Answers 2, 3: Circumferential casting at > 90 degrees may lead to compartment syndrome and closed reduction at lesser degrees is unlikely to be stable. Answer 5: Although crossed pin constructs have been demonstrated to be biomechanically most resistant to torsional stress, placement of medial sided pins risks injury to the ulnar nerve.

 

(SAE13PE.48) Figure A is the radiograph of an 11-year-old boy who fell directly on his left shoulder while riding a bicycle. He complained of pain in his left shoulder. An MRI scan of the injury is likely to reveal what findings? Review Topic

 

 

 

  1. Disruption of the coracoclavicular ligaments

  2. Dislocation of the acromioclavicular joint

  3. Impingement of the inferior fragment on the brachial plexus

  4. Intact periosteal sleeve inferiorly

 

PREFERRED RESPONSE 4

 

In children, fractures of the distal clavicle are almost always through the distal physis and adjacent metaphysis, and, consequently, the acromioclavicular joint is rarely dislocated. The coracoclavicular ligaments usually remain attached to the thick periosteum on the undersurface of the clavicle and are rarely damaged. Because of the thick periosteum and intact ligaments, these fractures are inherently stable and heal well with conservative treatment. In contrast, similar injuries in adults pose a higher risk for surgical intervention.

 

(OBQ14.171) A 14-year-old gymnast presents after a fall from the balance beam with a hyperextension injury to her left knee. She could ambulate with pain but was unable to continue exercise due to pain. On examination she has a swollen knee with painful

range of motion. AP and Lateral Xrays of the knee shown in Figures A and B. Treatment should consist of: Review Topic

 

 

 

  1. Immediate weight-bearing as tolerated with hinged knee brace 0-90 degrees

  2. Transphyseal ACL reconstruction with hamstring autograft

  3. Physeal spearing ACL reconstruction with hamstring autograft

  4. Application of long leg cast following successful fluoroscopic closed reduction

  5. Application of long leg cast for 1-2 weeks with transition to hinged knee brace 0-90 degrees, progressive weight-bearing, and physical therapy for quadriceps strengthening

 

PREFERRED RESPONSE 4

 

The patient has a mild to moderately displaced tibial eminence fracture, which can be treated with closed reduction, casting, and supportive care provided successful closed reduction is achieved.

 

Tibial eminence fractures are rare but occur more often in pediatric populations, often in the setting of sports-related injuries. Debate continues over operative vs nonoperative treatment, as well as fixation type (screw vs suture) for openly treated fractures. Past evidence suggested closed treatment was adequate but there has been an increase in operative management. Closed treatment is suggested for minimally displaced fractures (Type I and reducible Type II) and open treatment for completely displaced fractures (non-reducible Type II and Type III).

 

Wilfinger et al provide the results of a closed reduction protocol at their institution including 38 patients with long term followup. All patients underwent aspiration and closed reduction in the OR under fluoroscopic guidance followed by long leg casting in hyperextension and graduated weight bearing over weeks. No patients complained of persistent pain, swelling, giving way, or disability at follow up.

 

However, Edmonds et al in a retrospective review compare open reduction internal fixation (ORIF), arthroscopic-assisted internal fixation (AAIF), and closed reduction with casting (CRC) for pediatric patients with displaced tibial spine fractures. They report improved reduction but also increased arthrofibrosis in ORIF and AAIF groups

compared to CRC, but of the 24% of patients with long term followup results, there was no difference in functional outcomes across all 3 groups. There was a 17% rate of later operation for the CRC group patients. They suggest closed treatment for fractures with <5mm displacement, otherwise ORIF or AAIF.

 

Gans et al conducted a systematic review focused on the questions of open vs closed reduction, and screw vs suture fixation. The 26-article review found insufficient evidence to have any clear recommendations. They did find reduced laxity and improved range of motion for minimally displaced fractures that had an open reduction, and that completely displaced fractures treated nonoperative had higher rates of nonunion.

 

Figures A and B are AP and lateral knee radiographs demonstrating a moderately displaced (Meyers and McKeever Type II) tibial spine fracture in a skeletally immature patient.

 

Incorrect Responses

Answer 1: Immediate weight-bearing not recommended for fracture healing Answer 2, 3: ACL reconstruction is not indicated for this tibial spine fracture Answer 5: 1-2 weeks is insufficient for cast immobilization in a non-operative treatment protocol before transitioning to hinged-knee brace and physical therapy.

 

(SAE10PE.80) A 12-year-old girl is seen after tripping and twisting her ankle earlier in the morning. She had immediate pain and swelling and was unable to bear weight. Radiographs are shown in Figures 80a through 80c. Appropriate treatment should consist of which of the following? Review Topic

 

 

 

  1. Short leg cast with no weight bearing for the first 3 weeks

  2. Short leg cast with immediate weight bearing

  3. Long leg cast with no weight bearing for the first 3 weeks

  4. Open reduction and internal fixation, avoiding the physeal growth plate and joint

  5. CAM walker with immediate weight bearing

 

PREFERRED RESPONSE 4

 

Salter Harris III and IV fractures of the medial malleolus often contain a large cartilaginous portion, larger than the apparent ossified fragment seen on radiographs. Consequently, articular incongruity is common. As such, open reduction and internal fixation is often required. Growth disturbance and angular deformity are also common complications of Salter Harris III and IV fractures of the distal tibia.

 

(SAE07PE.53) A newborn girl with an isolated unilateral dislocatable hip is placed in a Pavlik harness with the hips flexed 100 degrees and at resting abduction. Figure 23 shows an ultrasound obtained 2 weeks later. What is the next step in management? Review Topic

 

 

 

  1. Reposition the harness to hold the hips in 70 degrees of abduction

  2. Closed reduction and arthrography under anesthesia

  3. Open reduction and a spica cast

  4. Continued harness treatment in the current position

  5. Spica cast

 

PREFERRED RESPONSE 4

 

The infant has a well-positioned hip in the Pavlik harness and treatment should be continued in the current position. The success rate is over 90% with the use of this device for a dislocatable hip. Ultrasound is a useful tool to confirm appropriate positioning of the cartilaginous femoral head during treatment. If the femoral head is

not reduced after 2 to 3 weeks in the harness, this mode of treatment should be abandoned. Forceful extreme abduction can cause osteonecrosis of the femoral epiphysis and should be avoided. Closed reduction, arthrography, and spica casting are indicated if the hip cannot be maintained in a reduced position with the harness.

 

(SAE08OS.107) Figure 36 shows the AP pelvic radiograph of a 26-month-old boy who has a limp. He has no significant medical history and no pain. What is the most appropriate treatment plan? Review Topic

 

 

 

  1. Physical therapy

  2. Application of a Pavlik harness

  3. Arthrogram and closed reduction with spica casting under anesthesia

  4. Femoral lengthening with a unilateral fixator

  5. Open reduction with femoral shortening, capsulorrhaphy, and a spica cast

 

PREFERRED RESPONSE 5

 

The radiograph reveals a complete hip dislocation. A Pavlik harness is an accepted method of treatment for a dislocated hip up until 6 months of age. Closed reduction and spica casting with an arthrogram is often successful in young children up to 1-1/2 years to 2 years of age. Open reduction, capsulorrhaphy, and a spica cast is commonly recommended for younger children when attempts at closed reduction have failed or when the child is older than 2 years of age. A femoral shortening is also generally recommended in children older than 2 years of age in an attempt to reduce the muscle/compressive forces on the reduced hip.

 

(SBQ13PE.38) Pavlik harness treatment is appropriate for which of the following?: Review Topic

  1. 4 year old with a diaphyseal femur fracture and a neuromuscular disorder with lower extremity spasticity

  2. 2 month old with a displaced spiral mid-diaphyseal femur fracture

  3. 9 month old with a diaphyseal femur fracture with <2cm shortening

  4. 9 month old with a diaphyseal femur fracture with >2cm shortening

  5. 4 year old with a diaphyseal femur fracture, closed head injury and chest trauma

 

PREFERRED RESPONSE 2

 

Treatment with Pavlik harness or spica cast are options for patients <6 months of age with diaphyseal femur fractures. Pavlik harness provides adequate pain control for the short time required for healing in this age group. Significant remodeling potential can be expected.

 

Because of the significant remodeling and quick healing, treatment of diaphyseal femur fractures in children younger than 6 months focuses on providing comfort and avoiding complications. Spica casting likewise results in stable union without longterm sequelae but minor skin complications are more common and some favor Pavlik treatment in this age group.

 

Kocher et al. provide the AAOS Clinical Practice Guideline for the treatment of pediatric diaphyseal femur fractures. Their recommendation for diaphyseal femur fractures in children less than 6 months is Grade C, based on Level IV evidence (one retrospective comparative study and one case series). They recommend that Pavlik and spica casting are both acceptable treatment options in this age group.

 

Flynn et al. review the management of pediatric femoral shaft fractures. They recommend Pavlik treatment for children 6 months of age or less in preference to a spica cast, sometimes supplemented with a simple splint. This avoids the skin complications of spica casting.

 

Incorrect answers:

Answer 1: Pavlik harness is contraindicated in irreducible or teratologic dislocations Answers 3, 4: Children greater than 6 months are more appropriately treated with other modalities

Answer 5: Prior failure of Pavlik treatment is a contraindication to continuing Pavlik treatment

 

(SAE10PE.35) A 10-month-old infant has no flexion at the elbows, mild flexion contractures at the wrist, a rigid clubfoot deformity on the left foot, and a rigid rocker bottom deformity on the right foot. Examination of the patient's hips reveals limited

abduction with 80 degrees of hip flexion/extension and full range of motion of the knees. A radiograph of the pelvis is seen in Figure 35. What is the most appropriate treatment for the patient's hip problem? Review Topic

 

  1. Preliminary skin traction followed by closed reduction under general anesthesia

  2. Immediate closed reduction under anesthesia

  3. Preliminary skeletal traction followed by closed reduction under general anesthesia

  4. Bilateral open reduction performed through a medial approach

  5. Bilateral open reduction performed through an anterior approach

 

PREFERRED RESPONSE 4

 

The patient has arthrogryposis. Szoke and associates performed open reduction through a medial approach on 40 hip dislocations in 26 patients with this condition and reported good results in 80% and fair results in 12%. Due to the stiffness associated with this disorder, closed reduction with or without skin or skeletal traction is not feasible. Open reduction through an anterior approach is reserved for older children.

 

(SAE08OS.149) Figures 51a and 51b show the radiographs of a 12-year-old boy obtained after an attempted closed reduction of an elbow injury in the emergency department. His motor examination is intact but he reports decreased sensation along the palmar aspect of his ring and little finger. What is the best treatment plan? Review Topic

 

 

 

  1. Closed reduction and percutaneous pinning

  2. Reconstruction of the ulnar collateral ligament using palmaris tendon autograft

  3. Open reduction and internal fixation

  4. Splinting for 1 week and observation of the nerve injury

  5. Ulnar nerve exploration and transposition

PREFERRED RESPONSE 3

 

The radiographs reveal an incarcerated medial epicondyle fracture apparently associated with an elbow dislocation. The incarcerated position of the fragment warrants open reduction and internal fixation. Due to the close proximity of the ulnar nerve, percutaneous pinning of a medial epicondyle fracture is not recommended. Ulnar nerve symptoms secondary to this fracture usually do not require a formal transposition.

 

(SBQ13PE.97) A 3-year-old boy presents to your office for evaluation. On physical examination, the patient has large calves and uses his hands to assist in rising from the floor. Laboratory results from the pediatrician reports notably high creatinine kinase of 43000 IU/L. What is the most likely inheritance pattern? Review Topic

 

  1. Autosomal recessive

  2. Autosomal dominant

  3. X-linked recessive

  4. X-linked dominant

  5. Mitochondrial inheritance

 

PREFERRED RESPONSE 3

 

Duchenne's muscular dystrophy (DMD) has an X-linked recessive inheritance pattern.

 

Duchenne's muscular dystrophy is characterized by a lack of dystrophin on muscle biopsy. Gower's sign is a typical physical exam finding (child uses hands to assist in rising from sitting), along with pseudohypertrophy of the calves. Notable laboratory values include elevated creatinine kinase (CK), typically above 25,000 IU/L.

 

Sussman et al. provide a comprehensive review of DMD, highlighting characteristic findings, including pseudohypertrophy of the calves (due to replacement of skeletal muscle with adipose tissue), Gower's sign, as well as CK levels typically above 25,000 IU/L. CK levels can aid in distinguishing between Becker's dystrophy which has levels less than 25,000 IU/L.

 

Figure A exhibits a muscle biopsy (calf) at low power of DMD; note the replacement of muscle with adipose tissue.

 

Incorrect answers:

Answers 1,2,4,5: DMD is not inherited via these modes

 

(SBQ13PE.47) Figure A shows an ultrasound of a 2 week old infant being evaluated for developmental dysplasia of the hip. Which of the labels depict the alpha angle, and what anatomic landmarks define this angle? Review Topic

 

 

 

  1. Angle A, ilium and cartilaginous acetabulum

  2. Angle A, ilium and bony acetabulum

  3. Angle B, ilium and cartilaginous acetabulum

  4. Angle B, ilium and bony acetabulum

  5. Angle B, bony acetabulum and acetabular labrum

 

PREFERRED RESPONSE 4

 

The alpha angle is subtended by (1) a line drawn down the ilium to a point where it intersects with the bony and cartilaginous confluence of the acetabulum and (2) a line drawn along the roof of the bony acetabulum.

 

The coronal flexion view (as depicted in figure A) of the infant hip is perhaps more familiar if rotated 90 degrees to view as one would view an anterior posterior radiograph of the pelvis. Acetabular development is evaluated primarily by the alpha angle, indicating the morphology of the developing acetabulum, and the percentage of the femoral head that is covered by the bony acetabulum. These are each readily apparent on ultrasound. Normal hips should have an alpha angle of at least 60 degrees by 6-8 weeks of age.

 

Harcke et al. provide a current concepts review on the state of ultrasound in the diagnosis and management of developmental dysplasia of the hip (as of 1991). They report, since that time, ultrasound has become the mainstay for diagnostic confirmation and management of DDH.

 

LeBa et al. performed a study to look at whether ultrasound screening would increase in effectiveness if targeted toward infants with established risk factors for developmental dysplasia of the hip and normal findings on physical examination. They found dynamic ultrasound evaluation showed developmental dysplasia in 7.8% of patients who had normal physical exams and led to a change in treatment in 8%. They conclude selective ultrasound screening in infants with risk factors and normal

findings on physical examination is effective and leads to more effective treatment.

 

Figure A shows an ultrasound exam of a 2 week old infant. The alpha angle is labeled 'B'. The beta angle is lableled 'A'. Illustration A is a diagram from Tachdjian's illustrating diagrammatically the elements of these angles.

 

Incorrect answers.

Answers 1 and 2: Angle A represents the beta angle. Answer 3: Angle B is correct but the angle described is the beta angle Answer 5: Angle B is correct but the two structures do not describe the alpha angle.

 

(SAE09FA.85) A toddler is brought in by his parents for evaluation of gait problems. Birth history and neurologic examination are unremarkable. After evaluating femoral torsion, tibial torsion, and foot contour, the diagnosis is excessive internal tibial torsion. The parents should be advised to expect which of the following outcomes? Review Topic

 

  1. Resolution by age 3 or 4 years without active treatment in most patients

  2. Resolution by age 8 or 9 years without active treatment in most patients

  3. Resolution with casting as the most effective treatment

  4. Resolution with bracing and shoe modification as the most effective treatment

  5. Resolution with surgery as the most effective treatment

 

PREFERRED RESPONSE 1

 

Excessive internal tibial torsion is a common cause of intoeing in toddlers. In most children, this resolves spontaneously by 3 to 4 years of age. Intoeing in elementary age children is usually the result of excessive femoral anteversion. Studies have shown that active intervention (casting, splinting, and shoe modifications) has no demonstrable effect on the natural history or resolution of tibial torsion. Surgery is rarely indicated in adolescents with severe internal tibial torsion that has not resolved and is resulting in cosmetic and functional problems.

 

(SAE10BS.99) Which of the following regions in the growth plate is commonly affected in a Salter-Harris type II injury? Review Topic

 

  1. Reserve zone

  2. Proliferative zone

  3. Hypertrophic zone

  4. Primary spongiosa

  5. Epiphyseal zone

 

PREFERRED RESPONSE 3

 

A type II injury consists of a fracture along the hypertrophic zone of the growth plate with an attached metaphyseal bony fragment. The hypertrophic zone is the metaphyseal fragment and is located on the compressive or concave side, whereas periosteum is torn on the tensile or convex side. The reserve and proliferative zones remain with the epiphysis and the circulation is usually preserved.

 

(SAE07PE.60) Figure 24 shows the sitting AP and lateral spinal radiographs of a nonambulatory 12½-year-old boy with Duchenne muscular dystrophy who is being evaluated for scoliosis. The lumbar curve from T12 to L5 measures 36 degrees, and the thoracic curve from T3 to T12 measures 24 degrees on the AP radiograph. He has 5 degrees of pelvic obliquity. His forced vital capacity is 45% of predicted for height and weight. What is the most appropriate treatment for the spinal deformity? Review Topic

 

 

 

  1. Posterior spinal fusion from T2 to L5 with segmental instrumentation

  2. Anterior spinal fusion from L1 to L4, followed by posterior spinal fusion from T2 to the sacrum with segmental instrumentation including iliac fixation

  3. Custom-molded spinal orthosis worn 23 hours per day until skeletal maturity

  4. A spinal orthosis until age 14 years, followed by posterior spinal fusion with segmental instrumentation

  5. Adapted wheelchair seating with a custom-molded back support to correct scoliosis and kyphosis

 

PREFERRED RESPONSE 1

Posterior spinal fusion is the treatment of choice for scoliosis in patients with Duchenne muscular dystrophy once they are no longer able to walk. This treatment improves quality of life and upright wheelchair positioning. Its effect on pulmonary function is less clear, as pulmonary function will continue to decline because of the underlying muscle disease. While bracing and wheelchair modifications may slow the progression of the curve, progression will continue. Surgical intervention at this stage does not have to include the pelvis, which, in general, is indicated in curves of greater than 40 degrees, and when pelvic obliquity is greater than 10 degrees. Fixation to the pelvis should also be considered in lumbar curves where the apex is lower than L1. Surgical treatment usually can be safely performed if the vital capacity is greater than 35%.

 

(SAE11OS.79) Which of the following conditions routinely requires early surgical intervention in patients with Marfan syndrome? Review Topic

 

  1. Kyphosis

  2. Ankle instability

  3. Protrusio acetabula

  4. Progressive scoliosis

  5. Pseudarthrosis of the tibia

 

PREFERRED RESPONSE 4

 

Marfan syndrome is a challenging disease for the orthopaedic surgeon. Most problems of joint laxity, acetabular protrusio, and minor scoliosis curves are treated nonsurgically. Pseudarthrosis of the tibia is not seen in Marfan syndrome; it is more common in patients with neurofibromatosis (NF-1). Treating kyphosis is risky for vertebral subluxation. Rapidly progressive scoliosis in immature patients is associated with higher surgical complications, but surgery is indicated. Overcorrection is associated with significant cardiovascular complications and should be avoided.

 

(SAE07PE.56) What is the primary indication for performing an arthroscopic synovectomy on a patient with hemophilia that is the result of factor VIII deficiency? Review Topic

 

  1. Joint pain with radiographic evidence of joint space narrowing

  2. Joint stiffness that has not improved with physiotherapy and bracing

  3. Recurrent joint bleeding despite optimal medical management

  4. Prophylaxis for joint preservation in severe hemophilia (factor VIII level <1%)

  5. Decreasing ambulatory endurance despite optimal medical management

PREFERRED RESPONSE 3

 

Improved medical management has changed musculoskeletal outcomes for individuals with hemophilia. Patients with severe hemophilia receiving prophylactic administration of factor VIII may never develop a target joint that requires further orthopaedic intervention. Patients with moderate hemophilia and those patients with severe hemophilia not receiving prophylactic treatment will still develop joints that have recurrent hemarthroses. When recurrent hemarthrosis continues despite optimal medical management, synovectomy is indicated. While synovectomy is predictable in its ability to decrease joint bleeding, it does not necessarily improve joint range of motion or prevent the development of hemophilic arthropathy over time. It will not reverse articular damage to the joint once it has developed.

 

(SAE13PE.91) A 13-year-old boy who has a history of a pituitary adenoma has an unstable unilateral slipped capital femoral epiphysis. What is the indication for prophylactic pinning of the contralateral, unslipped side? Review Topic

 

  1. Patient gender

  2. Patient age

  3. Presentation with an unstable slipped epiphysis

  4. Coexisting endocrine disorder

 

PREFERRED RESPONSE 4

 

Endocrine disorders post the highest risk for bilateral involvement, and prophylactic pinning of the uninvolved side is most often recommended. Risk of contralateral slippage is highest in the youngest patients. In a study by Riad and associates, all girls younger than age 10 and all boys younger than age 12 presenting with a unilateral slipped capital femoral epiphysis subsequently developed a contralateral slip. Initial presentation of an unstable slip has not been shown to be an independent risk factor for later contralateral slippage.